LSAT and Law School Admissions Forum

Get expert LSAT preparation and law school admissions advice from PowerScore Test Preparation.

 Administrator
PowerScore Staff
  • PowerScore Staff
  • Posts: 8927
  • Joined: Feb 02, 2011
|
#40801
Complete Question Explanation
(The complete setup for this game can be found here: lsat/viewtopic.php?t=14119)

The correct answer choice is (B)

This is a partial List question, and frankly a rather annoying one because of the intermittent selection of days: Tuesday, Wednesday, and Friday. Thus, although the answer choices list the three vehicles in what appears to be consecutive order, the days being referenced are actually not in direct sequence.

The key to this question is to understand how the placement of L or P on Tuesday, Wednesday, or Friday will affect the template that is in effect, and thus the various placement possibilities.

Answer choice (A): In this answer P is serviced on Tuesday and L is serviced on Friday; that leaves no vehicle available for servicing on Saturday. Hence, this answer choice is incorrect.

Answer choice (B) is the correct answer choice, and can occur under Template #1 (as shown in the setup discussion).

Answer choice (C): When L is serviced on Wednesday, then P must be serviced on Saturday (which occurs only under Template #2). However, in Template #2, S is serviced on Friday. This answer lists S as being serviced on Tuesday, which is impossible. Thus, this answer choice cannot be true and is incorrect.

Answer choice (D): When L is serviced on Wednesday, then P must be serviced on Saturday (which occurs only under Template #2). However, in Template #2, S is serviced on Friday. This answer lists H as being serviced on Friday, which is impossible. Thus, this answer choice cannot be true and is incorrect.

Answer choice (E): When L is serviced on Friday, then P must be serviced on Saturday (which occurs only under Template #2). However, in Template #2, S is serviced on Friday. Thus, there is a conflict over the vehicle to be serviced on Friday, and therefore this answer choice cannot be true and is incorrect.

Get the most out of your LSAT Prep Plus subscription.

Analyze and track your performance with our Testing and Analytics Package.